Last visit was: 26 Apr 2024, 04:41 It is currently 26 Apr 2024, 04:41

Close
GMAT Club Daily Prep
Thank you for using the timer - this advanced tool can estimate your performance and suggest more practice questions. We have subscribed you to Daily Prep Questions via email.

Customized
for You

we will pick new questions that match your level based on your Timer History

Track
Your Progress

every week, we’ll send you an estimated GMAT score based on your performance

Practice
Pays

we will pick new questions that match your level based on your Timer History
Not interested in getting valuable practice questions and articles delivered to your email? No problem, unsubscribe here.
Close
Request Expert Reply
Confirm Cancel
SORT BY:
Date
Tags:
Show Tags
Hide Tags
User avatar
Manager
Manager
Joined: 21 Jun 2014
Posts: 81
Own Kudos [?]: 361 [23]
Given Kudos: 59
Location: United States
Concentration: General Management, Strategy
GMAT 1: 630 Q45 V31
GPA: 3.4
WE:Engineering (Computer Software)
Send PM
Most Helpful Reply
Tutor
Joined: 16 Oct 2010
Posts: 14828
Own Kudos [?]: 64929 [8]
Given Kudos: 427
Location: Pune, India
Send PM
General Discussion
Senior Manager
Senior Manager
Joined: 08 Jun 2015
Posts: 259
Own Kudos [?]: 82 [0]
Given Kudos: 145
Location: India
GMAT 1: 640 Q48 V29
GMAT 2: 700 Q48 V38
GPA: 3.33
Send PM
avatar
Intern
Intern
Joined: 14 Oct 2015
Posts: 4
Own Kudos [?]: 2 [0]
Given Kudos: 3
Location: India
Concentration: Technology, Strategy
GMAT 1: 540 Q43 V22
GMAT 2: 700 Q47 V38
WE:Project Management (Computer Software)
Send PM
Re: The following argument took place at a recent town meeting [#permalink]
VeritasPrepKarishma wrote:
manishkhare wrote:
The following argument took place at a recent town meeting in Sun Grove, a well known tourist destination:

Local Newspaper Editor: Our town government continues to be fiscally irresponsible. Members of the town council have just approved over $120,000 in new salaries for three parking enforcement officers that the town simply cannot afford.

Town Mayor: Your argument is completely unfounded! Hiring new parking enforcement officers will help the town’s financial situation, not hurt it. Typically a parking enforcement officer brings in over twice what they are paid, so these three officers will surely have a net positive effect on the town budget in the future.

Which of the following exposes a flaw in the Town Mayor’s argument?

[A]There is a saturation point at which additional parking enforcement officers bring in slightly less than the officers hired before them.

[B]More parking enforcement typically incentivizes both tourists and locals to take public transportation, increasing profits on buses and trains owned by the town.

[C]Tourism is the primary revenue generator for the town of Sun Grove, and visitors rarely return to a tourist destination after receiving a ticket.

[D]Fines for parking are going to be increased dramatically, which usually results in fewer overall tickets being issued.

[E]Sun Grove has a policy that it will not take legal action if an individual has one unpaid parking ticket.


Press Kudos if you like the question.


Focus on what the Town Mayor says: "Hiring new parking enforcement officers will help the town’s financial situation, not hurt it. Typically a parking enforcement officer brings in over twice what they are paid, so these three officers will surely have a net positive effect on the town budget in the future."

Essentially, he is saying that the hiring of the officers will have a net positive effect on the budget in future.

Option (C) tells you that the hiring could actually have a negative effect on the future budget. More officers will mean more tickets being issued and that might discourage the tourists from coming in again. Hence, it weakens the mayor's argument since the hiring could have a net negative effect on the future budget.

The only other relevant options are (A) and (D)

[A]There is a saturation point at which additional parking enforcement officers bring in slightly less than the officers hired before them.
This tells you that the positive effect may be SLIGHTLY less. So it is not a flaw in the mayor's argument. The hiring could still have a net positive effect on the budget.

[D]Fines for parking are going to be increased dramatically, which usually results in fewer overall tickets being issued.
If parking fine amount will be increased, it will increase the revenue. If fewer tickets will be issued, it will reduce the revenue. Overall, we don't know the impact. Anyway, this has nothing to do with the hiring of three new enforcement officers and the effect that will have.

Hence answer (C)



Hi Karishma,

Are we not making additional assumption in Answer choice C such as "Tourists will sure be given tickets"?
Tutor
Joined: 16 Oct 2010
Posts: 14828
Own Kudos [?]: 64929 [0]
Given Kudos: 427
Location: Pune, India
Send PM
Re: The following argument took place at a recent town meeting [#permalink]
Expert Reply
psrehal wrote:
VeritasPrepKarishma wrote:
manishkhare wrote:
The following argument took place at a recent town meeting in Sun Grove, a well known tourist destination:

Local Newspaper Editor: Our town government continues to be fiscally irresponsible. Members of the town council have just approved over $120,000 in new salaries for three parking enforcement officers that the town simply cannot afford.

Town Mayor: Your argument is completely unfounded! Hiring new parking enforcement officers will help the town’s financial situation, not hurt it. Typically a parking enforcement officer brings in over twice what they are paid, so these three officers will surely have a net positive effect on the town budget in the future.

Which of the following exposes a flaw in the Town Mayor’s argument?

[A]There is a saturation point at which additional parking enforcement officers bring in slightly less than the officers hired before them.

[B]More parking enforcement typically incentivizes both tourists and locals to take public transportation, increasing profits on buses and trains owned by the town.

[C]Tourism is the primary revenue generator for the town of Sun Grove, and visitors rarely return to a tourist destination after receiving a ticket.

[D]Fines for parking are going to be increased dramatically, which usually results in fewer overall tickets being issued.

[E]Sun Grove has a policy that it will not take legal action if an individual has one unpaid parking ticket.


Press Kudos if you like the question.


Focus on what the Town Mayor says: "Hiring new parking enforcement officers will help the town’s financial situation, not hurt it. Typically a parking enforcement officer brings in over twice what they are paid, so these three officers will surely have a net positive effect on the town budget in the future."

Essentially, he is saying that the hiring of the officers will have a net positive effect on the budget in future.

Option (C) tells you that the hiring could actually have a negative effect on the future budget. More officers will mean more tickets being issued and that might discourage the tourists from coming in again. Hence, it weakens the mayor's argument since the hiring could have a net negative effect on the future budget.

The only other relevant options are (A) and (D)

[A]There is a saturation point at which additional parking enforcement officers bring in slightly less than the officers hired before them.
This tells you that the positive effect may be SLIGHTLY less. So it is not a flaw in the mayor's argument. The hiring could still have a net positive effect on the budget.

[D]Fines for parking are going to be increased dramatically, which usually results in fewer overall tickets being issued.
If parking fine amount will be increased, it will increase the revenue. If fewer tickets will be issued, it will reduce the revenue. Overall, we don't know the impact. Anyway, this has nothing to do with the hiring of three new enforcement officers and the effect that will have.

Hence answer (C)



Hi Karishma,

Are we not making additional assumption in Answer choice C such as "Tourists will sure be given tickets"?


It is logical that an increase in number of tickets given will result in an increase in number of tickets given to both locals and tourists. There is no reason to assume that tourists will not get more tickets too.
Intern
Intern
Joined: 29 Jan 2017
Posts: 33
Own Kudos [?]: 33 [0]
Given Kudos: 13
Send PM
Re: The following argument took place at a recent town meeting [#permalink]
Hi Karishma,

Are we not making additional assumption in Answer choice C such as "Tourists will sure be given tickets"?[/quote]

It is logical that an increase in number of tickets given will result in an increase in number of tickets given to both locals and tourists. There is no reason to assume that tourists will not get more tickets too.[/quote]

Is it not logical to think most tourists do not drive?
Manager
Manager
Joined: 14 Mar 2011
Posts: 133
Own Kudos [?]: 272 [0]
Given Kudos: 317
GMAT 1: 760 Q50 V42
Send PM
Re: The following argument took place at a recent town meeting [#permalink]
Yes, its perfectly logical to think this:
mrdlee23 wrote:
Is it not logical to think most tourists do not drive?


But, what we are doing here is exposing possible ways that will hurt the argument. Since, flaw and assumption are two sides of the same coin, negation do work very well for flaw too....:
Quote:
C. Tourism is the primary revenue generator for the town of Sun Grove, and visitors rarely do return to a tourist destination after receiving a ticket.
- does this support the argument ? Yes it does. It shows that the visitor, the main source of revenue, will return - thus strengthening the conclusion by eliminating other possible cause that may work against the prediction - "net positive effect on the town budget in the future".

The only problem I can see in this problem - C is more of a weakener than a flaw as it does provide an additional evidence and not a statement that is inherent to the logic of the argument.
Tutor
Joined: 16 Oct 2010
Posts: 14828
Own Kudos [?]: 64929 [0]
Given Kudos: 427
Location: Pune, India
Send PM
The following argument took place at a recent town meeting [#permalink]
Expert Reply
mrdlee23 wrote:
Hi Karishma,

Are we not making additional assumption in Answer choice C such as "Tourists will sure be given tickets"?

It is logical that an increase in number of tickets given will result in an increase in number of tickets given to both locals and tourists. There is no reason to assume that tourists will not get more tickets too.

Is it not logical to think most tourists do not drive?


There is no reason to assume that most tourists do not drive. In fact, in many cities, you do not have a good public transport system and hiring a car is the best option. In any case, even if say, only 10% tourists drive, considering that tourism is the main economic driver, it may actually lead to big numbers.
Intern
Intern
Joined: 11 Apr 2016
Posts: 13
Own Kudos [?]: 10 [0]
Given Kudos: 46
GMAT 1: 680 Q49 V34
GMAT 2: 710 Q50 V36
Send PM
Re: The following argument took place at a recent town meeting [#permalink]
VeritasPrepKarishma wrote:
mrdlee23 wrote:
Hi Karishma,

Are we not making additional assumption in Answer choice C such as "Tourists will sure be given tickets"?

It is logical that an increase in number of tickets given will result in an increase in number of tickets given to both locals and tourists. There is no reason to assume that tourists will not get more tickets too.

Is it not logical to think most tourists do not drive?


There is no reason to assume that most tourists do not drive. In fact, in many cities, you do not have a good public transport system and hiring a car is the best option. In any case, even if say, only 10% tourists drive, considering that tourism is the main economic driver, it may actually lead to big numbers.


Hi Karishma,

Newspaper editor is claiming that the decision to fiscally worth it. "fiscally" here doesn't specify which revenue we are particularly talking about her. I am going to assume that fiscally means revenue from tickets issued + revenue from tourists.

For C to be correct, we need to assume that returning visitors contribute significantly to the revenue and the additional revenue from ticketing will not offset the decrease in revenue from the non returning customers.

For A to be correct, i do agree with that if the ticketing revenue < the salary offered, the hired personnel will still be getting some +ve revenue, except if ticketing hurts revenue from tourists (as in C).

No option seems iron clad.
Math Expert
Joined: 02 Sep 2009
Posts: 92929
Own Kudos [?]: 619142 [0]
Given Kudos: 81609
Send PM
Re: The following argument took place at a recent town meeting [#permalink]
Expert Reply
manishkhare wrote:
The following argument took place at a recent town meeting in Sun Grove, a well known tourist destination:

Local Newspaper Editor: Our town government continues to be fiscally irresponsible. Members of the town council have just approved over $120,000 in new salaries for three parking enforcement officers that the town simply cannot afford.

Town Mayor: Your argument is completely unfounded! Hiring new parking enforcement officers will help the town’s financial situation, not hurt it. Typically a parking enforcement officer brings in over twice what they are paid, so these three officers will surely have a net positive effect on the town budget in the future.

Which of the following exposes a flaw in the Town Mayor’s argument?

[A]There is a saturation point at which additional parking enforcement officers bring in slightly less than the officers hired before them.

[B]More parking enforcement typically incentivizes both tourists and locals to take public transportation, increasing profits on buses and trains owned by the town.

[C]Tourism is the primary revenue generator for the town of Sun Grove, and visitors rarely return to a tourist destination after receiving a ticket.

[D]Fines for parking are going to be increased dramatically, which usually results in fewer overall tickets being issued.

[E]Sun Grove has a policy that it will not take legal action if an individual has one unpaid parking ticket.


Press Kudos if you like the question.


VERITAS PREP OFFICIAL SOLUTION:



C. In this argument, the editor concludes that the town government is fiscally irresponsible because it is planning on spending $120,000 that it cannot afford. The town mayor responds strongly that the editor is wrong – that spending the $120,000 will actually be good for the town’s fiscal situation. Why? Because the parking enforcement officers typically bring in over twice what they are paid and spending $120,000 will thus create a “net positive effect on the town budget in the future.” The correct answer will weaken the quoted conclusion in the previous sentence. (A) is tricky because if there is such a saturation point, then additional officers might not bring in over twice what they are paid. However, the correct answer needs to show that there will NOT be a net positive effect on the budget and nothing in (A) proves this. Answer choice (B) strengthens the argument: not only will the town get more money from the extra parking officers, but it will also now get more from public transportation. (C) is correct as it shows that the new parking enforcement officers could create a net negative effect on the budget: if tourism is the most important revenue generator for the budget, and extra tickets could drive down tourism, then the new parking officers will not necessarily have a net positive effect. In (D) the higher fines combined with fewer tickets might produce more revenue or less revenue. This is unknown and thus does not weaken the mayor’s position. For (E), legal action relating unpaid tickets does not weaken the mayor’s argument – you have no idea how this would affect revenue for the town.
Director
Director
Joined: 09 Aug 2017
Posts: 689
Own Kudos [?]: 415 [1]
Given Kudos: 778
Send PM
Re: The following argument took place at a recent town meeting [#permalink]
1
Kudos
Can anyone here just explain what is difference between finding a flaw and weakening the argument?
Here, question stem ask for a flaw but eventually we end up with weakening the Mayor's argument.

In my view, when question ask for a flaw, we have to find a kind of assumption which will negatively impact the argument.
Yes. Similar case is occurring in case C but C is bluntly weakening the argument.
User avatar
Non-Human User
Joined: 01 Oct 2013
Posts: 17227
Own Kudos [?]: 848 [0]
Given Kudos: 0
Send PM
Re: The following argument took place at a recent town meeting [#permalink]
Hello from the GMAT Club VerbalBot!

Thanks to another GMAT Club member, I have just discovered this valuable topic, yet it had no discussion for over a year. I am now bumping it up - doing my job. I think you may find it valuable (esp those replies with Kudos).

Want to see all other topics I dig out? Follow me (click follow button on profile). You will receive a summary of all topics I bump in your profile area as well as via email.
GMAT Club Bot
Re: The following argument took place at a recent town meeting [#permalink]
Moderators:
GMAT Club Verbal Expert
6921 posts
GMAT Club Verbal Expert
238 posts
CR Forum Moderator
832 posts

Powered by phpBB © phpBB Group | Emoji artwork provided by EmojiOne